- Sat Oct 21, 2017 9:20 am
#40796
Complete Question Explanation
(The complete setup for this game can be found here: lsat/viewtopic.php?t=4676)
The correct answer choice is (D)
The question stem stipulates that a WZ block exists. This can occur under either template, creating the following modified templates:
Answer choice (A) is incorrect because, as shown in Template #2, L could dive first.
Answer choice (B) is incorrect because T could dive third under either template:
Template #1 hypothetical: O-P-T-W-Z-L.
Template #2 hypothetical: L-P-T-W-Z-O.
Answer choice (C) is incorrect because, under Template #1, Z could dive third under this hypothetical: T-W-Z-O-P-L.
Answer choice (D) is the correct answer because P cannot dive fourth in either template; in both instances, placing P fourth would create problems in placing the WZ block.
Answer choice (E) is incorrect because Z could dive fourth under either template:
Template #1 hypothetical: O-T-W-Z-P-L.
Template #2 hypothetical: L-T-W-Z-P-O.
(The complete setup for this game can be found here: lsat/viewtopic.php?t=4676)
The correct answer choice is (D)
The question stem stipulates that a WZ block exists. This can occur under either template, creating the following modified templates:
Answer choice (A) is incorrect because, as shown in Template #2, L could dive first.
Answer choice (B) is incorrect because T could dive third under either template:
Template #1 hypothetical: O-P-T-W-Z-L.
Template #2 hypothetical: L-P-T-W-Z-O.
Answer choice (C) is incorrect because, under Template #1, Z could dive third under this hypothetical: T-W-Z-O-P-L.
Answer choice (D) is the correct answer because P cannot dive fourth in either template; in both instances, placing P fourth would create problems in placing the WZ block.
Answer choice (E) is incorrect because Z could dive fourth under either template:
Template #1 hypothetical: O-T-W-Z-P-L.
Template #2 hypothetical: L-T-W-Z-P-O.
You do not have the required permissions to view the files attached to this post.